LSAT and Law School Admissions Forum

Get expert LSAT preparation and law school admissions advice from PowerScore Test Preparation.

 Administrator
PowerScore Staff
  • PowerScore Staff
  • Posts: 8916
  • Joined: Feb 02, 2011
|
#33146
Complete Question Explanation

Strengthen—PR. The correct answer choice is (C)

The author of this letter to the editor expresses a conclusion that differs from the position apparently taken by the editor. It seems the editor previously stated that if the government were to confiscate a portion of the wages of convicted burglars when they reenter the workforce, then that confiscation would be a form of stealing, and would be an abuse of power by the government.

The letter author disagrees, citing a proposal now being considered. Under the proposal, the government would confiscate the wages of convicted burglars reentering the workforce in order to fund an account to compensate burglary victims. The author does not concede that such confiscation is stealing. However, the author concludes that, even if such confiscation were a form of stealing, the confiscation would still be justified.

This conclusion results from the application of some principle, or rule, that is not expressly stated in the stimulus. The failure to expressly make this rule part of the argument is a logical flaw. Your prephrase in this Strengthen question is that the correct answer choice will provide a rule supporting the notion that stealing from a person who burglarized others is justified, if the purpose of the theft is to compensate burglary victims. While the correct answer choice likely will not use these exact terms, it must perform the same role as the prephrase statement, namely providing a reason that supports the conclusion that the proposed confiscation is justified, even if the confiscation is a form of stealing.

Answer choice (A): This answer choice is incorrect because the conclusion dealt with the issue of whether confiscation of a convicted burglar’s wages, whether or not such confiscation is a form of stealing, is justified when the purpose of the confiscation is to fund an account to compensate burglary victims. Information stating that money stolen from a burglar should be given to that burglar’s victims does not address the issue of justification. This answer choice improperly addresses the distribution of the money, rather than whether it is justified to take the money from the burglar in the first place.

Answer choice (B): The proposal under consideration did not contemplate direct compensation by each convicted burglar to that burglar’s respective victims. Rather, the proposal was to fund an account for burglary victims generally. So, this principle does not apply to the conclusion.

Answer choice (C): This is the correct answer choice. While this answer choice does not justify the conclusion that the proposed confiscation is itself justified, it does provide at least some support for that position. The support provided by this answer choice derives from the rule that the motive prompting an action, in this case the desire to compensate burglary victims, determines whether an action, here the confiscation of wages from convicted burglars, is justified.

Answer choice (D): This answer choice is incorrect because it merely describes what is required for a crime to be justified, rather than providing support for the conclusion that this confiscation is justified.

Answer choice (E): The rule provided in this answer choice, if applied to the author’s argument, would invalidate the author’s conclusion to the extent it held that the proposal would be justified even if the proposed confiscation were a form of stealing.
 angie23
  • Posts: 25
  • Joined: Nov 17, 2013
|
#16794
I had issues with C) being the correct answer, particularly because of the phrase, "whether or not that action is justified." This doesn't really make it certain that the stimulus's argument that "Even if confiscating...would still be justified." Help?
 Nikki Siclunov
PowerScore Staff
  • PowerScore Staff
  • Posts: 1362
  • Joined: Aug 02, 2011
|
#16880
Hi angie23,

Here's what the author is saying: alright, confiscating convicted burglars' wages may be a form of stealing, but that's OK. Why? Because it is done in order to compensate their victims. The stem asks us to identify a principle that strengthens this line of reasoning.

Clearly, the author believes that the motive (to repay victims) behind an action (to confiscate a portion of the wages of convicted burglars) is relevant to whether the action is justified or not. Thus, answer choice (C) would strengthen the conclusion: the motive prompting the action determines whether or not that action is justified. I'm not sure why you are troubled by the mention of "whether or not": answer choice (C) would be equally correct if it said, "the motive prompting an action determines whether that action is justified." You may be overthinking this a tiny bit... :)

Does that make sense?
 angie23
  • Posts: 25
  • Joined: Nov 17, 2013
|
#16922
Not really. I couldn't see how anyone could make a jump from "whether or not that action is justified" to "it would be justified." Is the answer choice correct because it most helps the argument even though its not that strong? What am I missing here?
 Nikki Siclunov
PowerScore Staff
  • PowerScore Staff
  • Posts: 1362
  • Joined: Aug 02, 2011
|
#16935
Hi angie23,

You are partly correct: answer choice (C) does not stipulate that an action is always justified by its motive (if it said that, it would justify the conclusion). Instead, answer choice (C) merely states that the motive determines whether or not the action is justified, i.e. the motive is always relevant to, or has an effect on, whether the actions prompted by it are justified. If that were true, then using the motive behind the government's actions to justify them makes more sense. Answer choice (C) indeed leaves open the possibility that some motives may not justify the actions prompted by them. However, the question stem does not ask us to Justify the conclusion, but instead merely to strengthen it. Establishing that one's motives are always relevant to the justificatory logic behind their actions definitely strengthens the argument.

Thanks!
 JennuineInc
  • Posts: 18
  • Joined: May 11, 2016
|
#30297
Hi,

I was between A & C. Is A wrong because it doesn't explicitly say that its Justified? I thought A was a principle.

If money compensates victim :arrow: the stealing is justified.

A. Money stolen from a burglar should be given to that burglar's victims. Wouldn't that Justify the action of "stealing"?

Am I making an assumption on top of the answer?

I thought C was too vague because "the motive determines if the action is justified" doesn't tell me that the motive of "compensating burglary victims" is enough to conclude that the action is justified.

Thank you!
 15veries
  • Posts: 113
  • Joined: Sep 25, 2016
|
#30305
Why is B wrong?
I thought if they have obligation, this means the government's action will be justified because those money will go to compensate for the victim.
 Adam Tyson
PowerScore Staff
  • PowerScore Staff
  • Posts: 5153
  • Joined: Apr 14, 2011
|
#30628
The problem, 15, is that we don't know that an obligation makes the "stealing" justified. Imagine a slightly different scenario, wherein you owe your landlord some back rent. Would I be justified in breaking into your house and taking your TV, selling it, and then giving the money to your landlord? Answer choice B asks you to make the leap that it would be - that the very existence of your obligation makes it okay for another to steal from you to pay your debt.

Answer C makes the leap for us and tells us that motive matters. If that's so, then I might very well be justified in stealing your TV. Maybe not, but at least that principle helps us get there. Answer B doesn't, because it makes no connection between you being obligated and me being justified.

Be careful about bringing in your own ideas from outside the scope of the argument. You may have thought an obligation justified the stealing, but that didn't come from the answer choice - it came from your own ideas. Don't "help" the answer choices by bringing in your own outside ideas, but instead analyze them on their own merits.

Keep at it!
 menkenj
  • Posts: 116
  • Joined: Dec 02, 2020
|
#89229
Is D wrong because it is a reversal of the principle we're looking for?
User avatar
 lschlueter
  • Posts: 4
  • Joined: Jun 01, 2021
|
#89622
very confused why D dosen't work. apparently, answer only has to provide "some" support, and D does support that at least one necessary condition has been fulfilled. we ARE providing compensation to those that deserve it, so at least we are on our way to being justified. that does help. why does fulfilling a necessary condition help "less" than C, a rationale which could potentially cut equally for or against the argument (the motive could also not determine it, even given C).

Get the most out of your LSAT Prep Plus subscription.

Analyze and track your performance with our Testing and Analytics Package.